LSAT and Law School Admissions Forum

Get expert LSAT preparation and law school admissions advice from PowerScore Test Preparation.

 Administrator
PowerScore Staff
  • PowerScore Staff
  • Posts: 8917
  • Joined: Feb 02, 2011
|
#35062
Complete Question Explanation

Flaw—CE,#%. The correct answer choice is (C)

Your task in this Weaken question is to select the answer choice that most undermines the
researcher’s conclusion.

..... Premise: ..... every year approximately the same number of people die of iatrogenic
..... ..... ..... ..... “disease” —that is, as a direct result of medical treatments or
..... ..... ..... ..... hospitalization—as die of all other causes combined.

..... Conclusion: ..... if medicine could find ways of preventing all iatrogenic disease, the number of
..... ..... ..... ..... deaths per year would decrease by half

The flaw in the researcher’s argument is the assumption that if a person did not die of iatrogenic
“disease”, then the person would have lived. However, from the definition of iatrogenic disease, the
person already was ill or injured to a degree requiring medical treatment or hospitalization. Your
prephrase is that the conclusion does not consider the possibility that without hospitalization or
medical treatment, the person would have died anyway.

The incorrect answer choices will not undermine the conclusion, either because they have no effect
on the conclusion or they support it.

Answer choice (A): This choice is incorrect, because the idea of preventing noniatrogenic disease
was not relevant to the conclusion.

Answer choice (B): The argument did not distinguish between invasive or damaging medical
treatment and other medical treatments as a cause of iatrogenic disease, and so there is insufficient
information to conclude that it was a logical flaw not to discuss this alternative.

Answer choice (C): This is the correct answer choice. This answer is correct because it points out
the error in causal reasoning made by argument, in which the researcher assumes that preventing one
of many causes of death will prevent people from dying.

Answer choice (D): The researcher’s conclusion did not pertain to whether it is possible to prevent
all causes of death from iatrogenic disease, but rather the effect on the number of deaths per year if
iatrogenic disease was eliminated.

Answer choice (E): It appears the argument did not fail to consider the risk of iatrogenic disease
is present whenever there is a noniatrogenic disease, because the definition of iatrogenic disease is
so broad that it covers any situation in which not just hospitalization, but any medical treatment, is
present.
 mpoulson
  • Posts: 148
  • Joined: Mar 25, 2016
|
#24489
Hello,

I wanted to see if my logic was correct for why A is wrong. The answer would have had to say that the prevention of iatrogenic will lead to an increase in noniatrgenic deaths rather than the way it is written "prevention of nonitagenic disease will have an effect on the occurrence of iatrogenic disease. If it is wrong for another please let me know. Thank you.

- Micah
 Nikki Siclunov
PowerScore Staff
  • PowerScore Staff
  • Posts: 1362
  • Joined: Aug 02, 2011
|
#24622
Micah,

You're absolutely correct! Answer choice (A) is the Reverse answer; if you modify it the way you suggested, it would be a much better representation of the flaw.

Thanks,
 JennuineInc
  • Posts: 18
  • Joined: May 11, 2016
|
#26024
Hi,

Can someone please explain the answer for this question?

I understood the stimulus as even if medicine finds a way to prevent iatrogenic diseases then more people will die of the other causes because they're not getting hospital treatment.

Actually phrasing it that way just made me see why C is correct...
 Robert Carroll
PowerScore Staff
  • PowerScore Staff
  • Posts: 1787
  • Joined: Dec 06, 2013
|
#26025
Jennuine,

Although the stimulus is not saying that if people don't die of iatrogenic diseases, they will die of other causes, it's helpful to consider that possibility because it's something the stimulus ignores. Precisely the flaw is that the stimulus thinks if iatrogenic diseases were eradicated, the "slack" would not be taken up by other causes of death.

I think you know this now, though, so I'll say no more unless you have further questions!

Robert Carroll
 lsatstudier
  • Posts: 49
  • Joined: Oct 24, 2016
|
#31940
Hi,

These type of flaw questions tend to throw me for a loop. I identified the conditional reasoning in the conclusion and went on to look for the flaw type in the conclusion to find it was not there :0 How would you recommend approaching this type of question?

Thank you so much for all of your help with this!
 David Boyle
PowerScore Staff
  • PowerScore Staff
  • Posts: 836
  • Joined: Jun 07, 2013
|
#31973
lsatstudier wrote:Hi,

These type of flaw questions tend to throw me for a loop. I identified the conditional reasoning in the conclusion and went on to look for the flaw type in the conclusion to find it was not there :0 How would you recommend approaching this type of question?

Thank you so much for all of your help with this!

Hello lsatstudier,

It may not be clear what you mean by "I identified the conditional reasoning in the conclusion and went on to look for the flaw type in the conclusion to find it was not there". Do you mean that you saw there was conditional reasoning, so that you looked for a "sufficient-necessary" flaw among the answer choices? Or something else?
As for approaching flaw questions, try to look for what flaws they might have. Some problems in this question relate to cause and effect, and also numbers and percentages. So, you might look at the problem from a common-sense point of view and say, "Well, just because iatrogenic disease is prevented, maybe someone might die of a heart attack tomorrow anyway!" (Otherwise put: just because one cause of death is absent, that doesn't mean there couldn't be an extra, alternate cause of death.)
As for numbers, you might commonsensically note that there could be overlap between several spheres: just because iatrogenic disease causes roughly half of all deaths, that doesn't mean there's no overlap between iatrogenic and other spheres. Hypothetically, there could be 100% overlap! in that for all you know, every single person with iatrogenic problems would have died from something else a month later anyway! (Unlikely, but from the stimulus, you can't prove it's not true.)

Hope this helps,
David
 cardinal2017
  • Posts: 19
  • Joined: Oct 23, 2016
|
#33917
I was thinking between (c) and (e), then chose (E).

But now I got why (E) isn't correct although (c) and (e) seem similar.

Since this is a question which eventually makes you find a weakener,

(E) doesn't weaken the conclusion as much as (C) does.

As the conclusion provides, if medicine could find ways to prevent ALL iatrogenic diseases,

unlike (E) contends, it might be that 'a risk of iatrogenic disease tagging along a noniatrogenic must be not big of a deal since it can be prevented according to the provision of the conclusion.

Also, even if there might be 'a risk of iatrogenic' whenvever a noniatrogenic occurs, the number of deaths per year can still decrease (by half). So (E) doesn't destroy the conclusion as much as (C) does.'

By this post, I wanted to organize my initial confusion on this question between C and E and hope this helps someone else, too!
 Kristina Moen
PowerScore Staff
  • PowerScore Staff
  • Posts: 230
  • Joined: Nov 17, 2016
|
#33925
Hi cardinal,

Nice job reviewing your work. But I would add that answer choice (E) doesn't really weaken the conclusion at all. The conclusion is "Therefore, if medicine could find ways of preventing all iatrogenic disease, the number of deaths per year would decrease by half." So knowing what causes iatrogenic disease does not weaken the conclusion because we are imagining a world where iatrogenic disease is totally prevented. It's sort of like if I said "If unicorns existed, they would be hunted for their horns." and you responded with "But unicorns don't exist." That still wouldn't weaken my conclusion because my conclusion is reliant on the "if" statement.

This is basically a numbers game. The conclusion is about decreasing the number by half. Say 2 people die from non-iatrognic disease and 2 people die from iatrogenic disease every year. If we eliminate iatrogenic disease, then 2 people won't die from iatrogenic disease and we've eliminated death by half, right? Not so fast, those 2 people could die a few months later in a car accident. Or by being gored by a unicorn. :-D So we can't be dead sure (no pun intended) that the number of deaths would decrease by half.
 ninamichelle
  • Posts: 5
  • Joined: Jul 19, 2017
|
#38915
I'm having some trouble with this question - I can see why C it is correct, and that it actually does lineup with my pre-phrase (which was something along the lines of, 'the researcher fails to consider that those who have contracted an iatrogenic disease are more likely than the general population to be in the hospital for treatment of a potentially deadly noniatrogenic disease"). My issue is that this language feels so broad and wishy-washy I eliminated it almost immediately. To say "just because you didn't die of one thing today doesn't mean you won't die of another thing tomorrow" seems like such a broad conception of the pre-phrase I identified above that I simply did not seriously consider it. Using a universal truth to weaken an argument does not strike me as a typical line of attack for LSAT authors...I guess I am just worried that I would, on test day, throw an answer phrased this broadly away. Is this type of language typical? How can I prepare myself for seeing an answer choice like this?

Get the most out of your LSAT Prep Plus subscription.

Analyze and track your performance with our Testing and Analytics Package.